LSAT and Law School Admissions Forum

Get expert LSAT preparation and law school admissions advice from PowerScore Test Preparation.

 Administrator
PowerScore Staff
  • PowerScore Staff
  • Posts: 8917
  • Joined: Feb 02, 2011
|
#36269
Complete Question Explanation

Flaw in the Reasoning. The correct answer choice is (E)

The stimulus argues that it was unfair to install speed bumps in Grove Park, since all drivers have the
right to use the public roads in Grove Park whenever they wish.

You probably had the strong commonsense reaction that speed bumps prevent speeding through a
neighborhood but do not prevent driving through it. That reaction is very helpful because it identififi es
a key fl aw in the argument, and the question stem asks you to fi nd this fl aw. The challenge then
becomes to fi nd the answer choice that best refl ects the author’s erroneous assumption that speed
bumps prevent the use of a road.

Answer choice (A): Although the argument ignores the possibility that speed bumps may not reduce
the speed at which drivers drive through the neighborhood, this is not a fl aw in the argument and so
this answer is incorrect. Rather, this answer describes a fl aw in the neighborhood’s traffi c control
plan (and again, not a fl aw in the author’s argument).

Answer choice (B): Ignoring that drivers speed through the neighborhood is not the error refl ected
in the stimulus. In fact, the reason for putting up the bumps is actually extraneous to the reasoning in
the argument since the argument takes the position that the speed bumps—whatever their intended
purpose—are unfair since they deprive drivers of the right to use the roads.

Answer choice (C): The issue in the stimulus is one of rights and road use, not whether drivers have
actually complained. So, although the stimulus does not present any information about complaints,
there is no need to do so.

Whether or not those who still drive through the neighborhood complain about the speed bumps, the
speed bumps could still be unfair, especially to those who quit using the road, so this response does
not express a fl aw in the argument.

Answer choice (D): Actually, the stimulus assumes that the residents of communities do not have any
such right, so this response is an Opposite answer, and is incorrect.

Answer choice (E): This is the correct answer choice. The argument does not prove that the speed
bumps prevent usage of the neighborhood’s roads, but the argument depends on the assumption that
they do.

Get the most out of your LSAT Prep Plus subscription.

Analyze and track your performance with our Testing and Analytics Package.